A bicycle shop rent bicycles by the hour. It charges and equipment fee with every rental. The graph shows the cost of renting a bicycle for different periods of time. Drag numbers to complete a linear function in terms of why equals MX plus E tomorrow the situation. Numbers may be used once, more than once, or not at all.

A Bicycle Shop Rent Bicycles By The Hour. It Charges And Equipment Fee With Every Rental. The Graph Shows

Answers

Answer 1

y = 15x + 10

Explanation:

Given a graph, we use the equation of line y = mx + b

m = slope

b = y-intercept

First let's find the slope using the formula:

[tex]m\text{ = }\frac{y_2-y_1}{x_2-x_1}[/tex][tex]\begin{gathered} u\sin g\text{ any two points on the line: } \\ (2,40)\text{ and (4, 70)} \\ x_1=2,y_1=40,x_2=4,y_2\text{ = }70 \\ m\text{ = slope = }\frac{70-40}{4-2} \\ \text{slope = }\frac{30}{2}\text{ = 15} \end{gathered}[/tex]

The y-intercept is the value of y when x = 0. It is also the value of y when the line crosses the y-axis.

The value of cost when time is zero

The line crosses the y-axis at y = 10

Hence, the y-intercept is 10

The equation of line becomes:

y = 15x + 10


Related Questions

Consider the function g. 9(-) = 6() For the x-values given in the table below, determine the corresponding values of g(x) and plot each point on the graph.. -1 0 1 2 g(x) Drawing Tools Click on a tool to begin drawing * Delete Undo Reset Select Point 14 13 12 11 10 9 00 reserved.

Answers

we have the function

[tex]g(x)=6(\frac{3}{2})^x[/tex]

Find out the value of function g(x) for each value of x

so

For x=-1

substitute the value of x in the function g(x)

[tex]\begin{gathered} g(-1)=6(\frac{3}{2})^{-1} \\ g(-1)=6(\frac{2}{3}) \\ g(-1)=4 \end{gathered}[/tex]

For x=0

[tex]\begin{gathered} g(0)=6(\frac{3}{2})^0 \\ g(0)=6 \end{gathered}[/tex]

For x=1

[tex]\begin{gathered} g(1)=6(\frac{3}{2})^1 \\ g(1)=9 \end{gathered}[/tex]

For x=2

[tex]\begin{gathered} g(2)=6(\frac{3}{2})^2 \\ g(2)=13.5 \end{gathered}[/tex]

using a graphing tool

plot the different points

so

we have

(-1,4)

(0,6)

(1,9)

(2,13.5)

see the attached figure to better understand the problem

please wait a minute

Directons: Write each equation in slope-intercept form. Identify the slope and y-intercept.

Answers

Given:

The equation is x - y = -8.

Explanation:

The slope intercept form of linear equation is,

[tex]y=mx+c[/tex]

Here, m is slope and c is y-intercept.

Simplify the given equation to obtain in slope-intercept form.

[tex]\begin{gathered} x-y=-8 \\ y=x+8 \\ y=1\cdot x+8 \end{gathered}[/tex]

So slope of line is m = 1 and y-intercept is 8.

Answer:

Equation in slope ntercept form: y = x + 8

Slope: 1

Y-intercept: 8 OR (0,8)

You are given two overlaying squares with side length a. One of the squares is fixed at the
bottom right corner and rotated by an angle of α (see drawing). Find an expression for the
enclosed area A(α) between the two squares with respect to the rotation angle α.

Answers

The expression for the area enclosed between the two squares with respect to the rotation angle α is

(α/90)a².

What is a square?

A Square is a two-dimensional figure that has four sides and all four sides are equal.

The area of a square is given as side²

We have,

Side of the square = a

Area of the square = a²

The full angle that can be rotated is 90°.

Now,

The area enclosed if the angle is 90°.

= a²

We can write as,

The area enclosed in terms of the angle.

= (angle rotated / 90) x side²

= (angle rotated / 90) x a²

Now,

The angle rotated is α.

The area enclosed is (α/90)a².

Thus,

The expression for the area enclosed between the two squares with respect to the rotation angle α is

(α/90)a².

Learn more about squares here:

https://brainly.com/question/22964077

#SPJ1

Which fraction and decimal forms match the long division problem? 15) 4.000 301 1 00 90 100 90 A. and 0.26 15 В. 15 and 0.26 C. and 0.26 15 15 and 0.266

Answers

Which fraction and decimal forms match the long division problem? 15) 4.000 301 1 00 90 100 90 A. and 0.26 15 В. 15 and 0.26 C. and 0.26 15 15 and 0.266​

we have

4/15=0.26666

so

the answer is

4/15 and 0.26option A

Select the point that satisfies y≤ x²-3x+2.

Answers

The point A (4, 4) satisfies the equation y≤ x²-3x+2.

To check for the equation, substitute each point into the inequality and check validity of solution

A (4, 4)

4 ≤ 16 - 12 + 2 ⇒ 4 ≤ 6 → True hence valid solution

B (3, 3 )

3 ≤ 9 - 9 + 2 ⇒ 3 ≤ 2 → False hence not valid

C (1, 1 )

1 ≤ 1 - 3 + 2 ⇒ 1 ≤ 0 → False hence not valid

D (2, 2 )

2 ≤ 4 - 6 + 2 ⇒ 2 ≤ 0 → False hence not valid

Therefore, the point A (4, 4) satisfies the equation y≤ x²-3x+2.

To learn more about equation refer here

https://brainly.com/question/22688504

#SPJ9

Disclaimer: The question given by you is incomplete, the complete question is

Select the point that satisfies y ≤ x² - 3x + 2.

A. (4, 4)

B. (3, 3)

C. (1, 1)

D. (2, 2)

Answer:

The point A (4, 4) satisfies the equation y≤ x²-3x+2.

Step-by-step explanation:

Translate the following into algebraic equation and solve: Twice the sum of a number and five is equal to 40.

Answers

Let:

x = Unknown number

Twice the sum of a number and five:

[tex]2(x+5)[/tex]

Is equal to 40:

[tex]2(x+5)=40[/tex]

Solve for x:

Expand the left hand side using distributive property:

[tex]2x+10=40[/tex]

Subtract 10 from both sides:

[tex]2x=30[/tex]

Divide both sides by 2:

[tex]x=15[/tex]

Find the simplified product.3-532 + 3823B-5ob+32B + 32+325 + 6

Answers

Answer:

[tex]\frac{b+3}{2}[/tex]

Explanation:

Given the below expression;

[tex]\frac{b-5}{2b}\times\frac{b^2+3b}{b-5}[/tex]

Let's go ahead and simplify the expression as shown below;

[tex]\frac{b-5}{2b}\times\frac{b(b+3)}{b-5}=\frac{b(b+3)}{2b}=\frac{b+3}{2}[/tex]

Find the future value$4013 invested for 9 years at 4.1% compounded quarterly.

Answers

We are to find the future value

The future value can be calculated using

[tex]FV=PV(1+\frac{r}{100\alpha})^{n\alpha}[/tex]

From the given information

PV = $4013

r = 4.1

n = 9 years

Since the investment is compounded quarterly then

α = 4

By substituting these values we get

[tex]FV=\text{ \$4013(1 }+\frac{4.1}{100(4)})^{9(4)}[/tex]

Simplifying the equation we get

[tex]\begin{gathered} FV=\text{ \$}4013(1\text{ }+\frac{4.1}{400})^{36} \\ FV=\text{ \$}4013(1\text{ }+0.01025)^{36} \\ FV=\text{ \$}4013(1.01025)^{36} \\ FV=\text{ \$}4013(1.44436) \\ FV=\text{\$}5793.17 \end{gathered}[/tex]

Therefore,

The Future Value is $5793.17

need help asappppppp

Answers

Function, in mathematics, an expression, rule, or law that defines a relationship between one variable (the independent variable) and another variable (the dependent variable)

For the ordered pairs given, we have inputs and outputs as shown below

The values of the input should be unique

Checking all the options given,

The pair (4,2) when added to the pairs will not make the relation a function because

4 will have two

the points (v,-3) and (8,5) fall on a line with a slope of -8. what is the value of v?

Answers

The slope m is given by:

[tex]\begin{gathered} m=\frac{y2-y1}{x2-x1} \\ \text{Where:} \\ (x1,y1)=(v,-3) \\ (x2,y2)=(8,5) \\ m=-8 \\ so\colon \\ -8=\frac{5-(-3)}{8-v} \\ \text{solve for v:} \\ -8(8-v)=5+3 \\ -64+8v=8 \\ 8v=72 \\ v=\frac{72}{8} \\ v=9 \end{gathered}[/tex]

1. Write a function V(x) that models the volume of the box where the length of the sides of the squares is x cm. (The formula for the volume of a box is: V = l ⋅ w ⋅ ℎ).2. Graph V(x). (You may use Desmos or draw in the provided grid.)

Answers

From the problem, the length and the width will be reduced by twice the side of the square.

The length of the box will be :

[tex]26-2x[/tex]

The width of the box will be :

[tex]20-2x[/tex]

and the height will be the measurement of the square side :

[tex]x[/tex]

Note that the volume of a box is length x width x height.

1. The volume will be :

[tex]V(x)=x(26-2x)(20-2x)[/tex]

Expand and simplify the function :

[tex]\begin{gathered} V(x)=x(26-2x)(20-2x) \\ V(x)=x(520-40x-52x+4x^2) \\ V(x)=x(4x^2-92x+520) \\ V(x)=4x^3-92x^2+520x \end{gathered}[/tex]

2. Graph the function using desmos.

Lolslslsoosodidododdjsjjejds

Admission to the fair costs $6.00. Each ride costs you$0.50. You have $22.00 to spend at the fair on rides and admission. Express the number of tickets you can buy as an inequality.

Answers

Let:

x = Number of rides

Total money spent = $6.00 + $0.50x

Since you have $22.00 to spend at the fair on rides and admission:

[tex]\begin{gathered} 6+0.5x\leq22 \\ \text{solving for x:} \\ 0.5x\leq22-6 \\ 0.5x\leq16 \\ x\leq\frac{16}{0.5} \\ x=32 \end{gathered}[/tex]

Find the circumference of the circle. Give the exact circumference and then an approximation. Use i 3.14. diamater of 17cm

Answers

To find the circumference of the circle, we will follow the steps below

Formula for the circumference of a circle is

C = 2 π r

where C = circumference of the circle

π is a constant

r is the radius of the circle

From the question, diameter is 17 cm

radius is half the diameter

That is:

radius = 17/2 = 8.5 cm

π = 3.14

Substituting the parameter in the formula given will yield

C = 2 x 3.14 x 8.5 cm

C =53.38 cm

The exact circumference is 53.38 cm

The circumference is approximately 53 cm to the nearest whole number

what's the solution to this system

Answers

Remember that

when solving a system by graphing, the solution is the intersection point both graphs

so

In this problem

the intersection point is (-2,2)

therefore

the solution is (-2,2)

Need answer to pictured problem! The answer should be in reference to trig identities

Answers

Step 1. The expression that we have is:

[tex]cos^2(5x)[/tex]

and we need to find the equivalent expression.

Step 2. The trigonometric identity we will use to solve this problem is:

[tex]cos^2A=1-sin^2A[/tex]

In this case:

[tex]A=5x[/tex]

Step 3. Applying the trigonometric identity to our expression, substituting 5x in the place of A:

[tex]cos^2(5x)=\boxed{1-sin^2(5x)}[/tex]

This is shown in option d).

Answer:

[tex]\boxed{d)\text{ }1-s\imaginaryI n^2(5x)}[/tex]


Find the value of r so that the line through (-4, r) and (-8, 3) has a slope of -5.

Answers

The value of r is -17.

Given,

Points (-4,r) and (-8,3)

slope=-5

Let

A(x1,y1)=(-4,r)

B(x2,y2)=(-8,3)

To find 'r' use formula,

[tex]slope=\frac{y2-y1}{x2-x1}\\ \\-5=\frac{3-r}{-8-(-4)}\\\\-5=\frac{3-r}{-8+4}\\\\-5=\frac{3-r}{-4}\\\\20=3-r\\\\20-3=-r\\\\17=-r\\\\-17=r[/tex]

Thus, the value of r is -17.

To learn more about slope of line refer here

brainly.com/question/18298097

#SPJ9

According to the graph of H(w) below, what happens when w gets very large?H)5.6.20.00)A. H(w) gets very large.B. H(w) approaches a vertical asymptote.C. H(w) equals zero.D. H(w) gets very smallSUBMIT

Answers

Considering the graph H(w),

As w gets larger, H(w) continues to approach a horizontal asymptote.

Hence, H(w) gets very small.

Therefore, the correct option is option D

100 POINTS!! I NEED THIS KNOWW!!!!The number line shows the distance in meters of two birds, A and B, from a worm located at point X:A horizontal number line extends from negative 3 to positive 3. The point labeled as A is at negative 2.5, the point 0 is labeled as X, and the point labeled B is at 2.5.Write an expression using subtraction to find the distance between the two birds.Show your work and solve for the distance using additive inverses.

Answers

The expression used to represent the distance between the two birds on the number line is 2.5 - (-2.5) and the distance is 5 units.

Let us represent the bird A is sitting at one point on the number line.

Bird B is sitting at another point.

Both of them are at a an equal distance from a worm which is at the position O which is the origin.

Now it is given that A and B are at the positions of the number line marked 2.5 and -2.5

Now the distance between A and B can be calculated by finding the distance between A and O and adding the additive inverse to it to get the distance between O and B using subtraction.

AO = 2.5 - 0 = 2.5 units

The additive inverse of this is  -2.5 units.

Therefore the distance AB :

= AO - BO

= 2.5 -(-2.5)

=2.5 +2.5

=5 units.

Hence they are at a distance of 5 units from each other.

To learn more about number line visit:

https://brainly.com/question/20887529

#SPJ9

In quadrilateral ABCD, MZA = 72, mZB = 94, and m2C = 113. What is m2D?

Answers

First, let's picture the problem

Let's label the angle D as x

Remember that the sum of angles in a quadrilateral is 360

[tex]\begin{gathered} 72^0+94^0+113^0+x=360^{\square} \\ x=81^0 \\ m\angle D=81^0 \end{gathered}[/tex]

The boxplot below shows salaries for Construction workers and Teachers.ConstructionTeacher2025465030 35 40Salan (thousands of S)If a person is making the median salary for a construction worker, they are making more than what percentage ofTeachers?They are making more than% of Teachers.Check Answer

Answers

Const Workers , Teachers

Median salary of const Worker =45

Median salary of teacher = 40

Then, they are making more than 100% of teachers

Answer is

100%

3andLet's compare38ロ<ロ>=First, write the fractions with the same denominator.х?138-138Then, use <, = , or > to compare the fractions.m 100

Answers

To rewrite the fractions as fractions with the same denominator we have to determine the minimum number greater than 8 and 3 that can be exactly divided by 8 and 3 (LCM). Notice that the LCM of 8 and 3 is

[tex]24=8\cdot3.[/tex]

Because:

[tex]\begin{gathered} 8=2\cdot2\cdot2, \\ 3=3. \end{gathered}[/tex]

Therefore, we rewrite the given fractions as:

[tex]\begin{gathered} \frac{1}{3}=\frac{8}{24}, \\ \frac{3}{8}=\frac{9}{24}\text{.} \end{gathered}[/tex]

From the above fractions, we get that:

[tex]\frac{3}{8}>\frac{1}{3}\text{.}[/tex]

Answer:

a)

[tex]\begin{gathered} \frac{1}{3}=\frac{8}{24}, \\ \frac{3}{8}=\frac{9}{24}\text{.} \end{gathered}[/tex]

b)

[tex]\frac{1}{3}<\frac{3}{8}\text{.}[/tex]

For this problem identify P, FV, I, r, n, and t.

Answers

Answer:

a) P = 180,000cents

b) FV = 298,418cents

c) I = 118,418cents

d) r = 0.003375

e) n = 12

f) t = 15 years

Explanations:

a) P is the principal in the given question. The principal can be the amount invested or borrowed.

According to the question, the amount invested is $1,800.00, hence the principal P is $1,800 which is equivalent to 180,000cents to the nearest cents

b) The future value is the amount after 15 years of investing the money. According to the question, Sandra earned a total interest of $1,184.18 after 15 years.

Future value = Principal + Interest

Future value = $1,800.00 + $1,184.18

Future value = $2,984.18

Hence FV to the nearest cent is 298,418cents

c) Given the total interest of $1,184.18

Convert to nearest cents

I = 1,184.18 * 100

I = 118,418cents

d) r is the rate in percentage. From the question the rate in percent is

3 3/8 %

Convert to decimal

[tex]\begin{gathered} r=3\frac{3}{8}\% \\ r=\frac{27}{8}\% \\ r=\frac{27}{800} \\ r=0.03375 \end{gathered}[/tex]

e) n is the time of compounding. From the question, we are told that amount invested was compounded monthly. Since there are 12 months in a year, hence the value of n is 12.

f) "t" is the time taken by Sandra to invest $1,800 to earn the given interest. From the question, the time it takes is 15 years. Hence;

t = 15 years

1. Solve: 12 + 24 = 6 x 3 = ?

Answers

Answer:

9

Step-by-step explanation:

12 + 24 = 36

6 * 3 = 18

36 = 18 = ?

Well, 18 is 1/2 of 36, so the next sequence would be 1/2 of 18, which is 9.

I'm not quite sure that this would be correct, just because I have no more context.

if you shift the function F(x) = log10 x up four units, what is the new function, G(x)?*PHOTO*

Answers

Given:

The function

[tex]F(x)=log_{10}x[/tex]

Required:

If you shift the function up for four units. What is the new function G(x)?

Explanation:

We have that function is shifting up for four units that is on y axis.

So, the new function will look like

[tex]G(x)=log_{10}x+4[/tex]

Answer:

option A is correct.

What is the correct classification of the system of equations below?14x + 2y = 10y + 7x = -5A. parallelB. coincidentC. intersecting

Answers

Given:

14x + 2y = 10

y + 7x = -5

Required:

To tell which option is correct

Explanation:

14x + 2y = 10

y + 7x = -5

the given two lines intersect each other

Required answer:

Option C

If f -1(x) = (6/5)x - 9, find f (x).

Answers

Solution

Step 1

Write the inverse function:

[tex]f^{-1}(x)\text{ = }\frac{6}{5}x\text{ - 9}[/tex]

Step 2

[tex]\begin{gathered} Let\text{ f}^{-1}(x)\text{ = y} \\ \\ y\text{ = }\frac{6}{5}x\text{ - 9} \\ \\ Make\text{ x the subject of the formula} \\ \\ y\text{ + 9 = }\frac{6}{5}x \\ \\ Divide\text{ both sides by }\frac{6}{5} \\ \\ x\text{ = }\frac{5}{6}(y\text{ + 9\rparen} \\ \\ f(x)\text{ = }\frac{5}{6}(x\text{ + 9\rparen} \end{gathered}[/tex]

Final answer

[tex]f(x)\text{ = }\frac{5}{6}(x\text{ + 9\rparen}[/tex]

IIIDECIMALSRounding decimalsRound 0.434 to the nearest hundredth.0x

Answers

Answer

Explanation

In rounding off numbers, when the number after the required level of precision is less than 5, we round it down. But if that number is 5 or more, we round it up.

-2(y+5)+21<2(6-y) Solving for y

Answers

[tex]undefined[/tex]

For the rotation -1046°, find the coterminal angle from 0° < O < 360°, the quadrant and the reference angle.

Answers

Solution

Step 1

In order to find a coterminal angle, or angles of the given angle, simply add or subtract 360 degrees of the terminal angle as many times as possible.

Step 2

The reference angle is the smallest possible angle made by the terminal side of the given angle with the x-axis. It is always an acute angle (except when it is exactly 90 degrees). A reference angle is always positive irrespective of which side of the axis it is falling.

Coterminal angle

[tex]\begin{gathered} Coterminal\text{ angle = -1046 + 3}\times360 \\ Coterminal\text{ angle = -1046 + 1080} \\ Coterminal\text{ angle = 34} \end{gathered}[/tex]

Quadrant = 1st quadrant

Reference angle

0° to 90°: reference angle = angle

Reference angle = 34

Final answer

Find an equation for the line that’s passes through the following points shown in the picture. ( Please fins answer in timely answer very brief explaination :) )

Answers

The general equation of line passing through the points (x_1,y_1) and (x_2,y_2) is,

[tex]y-y_1=\frac{y_2-y_1}{x_2-x_1}(x-x_1)[/tex]

Determine the equation of line passing thgrough the point (-6,-1) and (2,5).

[tex]\begin{gathered} y-(-1)=\frac{5-(-1)}{2-(-6)}(x-(-6)) \\ y+1=\frac{6}{8}(x+6) \\ y+1=\frac{3}{4}x+\frac{9}{2} \\ y=\frac{3}{4}x+\frac{9}{2}-1 \\ =\frac{3}{4}x+\frac{7}{2} \end{gathered}[/tex]

So equation of line is y = 3/4x + 7/2.

Other Questions
Hello I need help with this problem for my home work heres a picture What is the phenotype of a black cat?A. blackB. bbC. BB or BbD. bb what is the mass of aluminum used if o.1 moles of aluminum chloride is produced by aluminum chloride reaction? Triangle ABC has vertices A(1,3), B(2,5), and C(5,3). What are the coordinates of B after the translation described by the rule T(1,4)?B = WILL GIVE BRAINLIEST AND NEED QUICKLY!!!Was B. F. skinner vehemently anti Darwinian?True or False What was the major reason why the French to join the AmericanRevolution Geometry!! Any help appreciated! Thank you 27. The following chart represent the cost, Cd), in dollars, of a pizza in relation to its diameter, d, in inches. What is the average rate of change in cost from an 8 in to a 16 in pizza. d Cd) 8 2.51 103.93 125.65 147.70 16 10.05 At a certain company, loan agents are paid based on the number of loans they close in a day. Based on company records, the number of loans X that arandomly selected loan agent closes on a randomly selected day has the probability distribution below.45 6P(x) 0.05 0.10 0.22 0.30 0.18 0.12 0.03X123At the company, the daily salary of a loan agent is $150 plus $50 per loan closed. Let Y represent the amount of money made by a randomly selected loanagent on a randomly selected day.Which is the mean of X?Which is the mean of Y?VWhich is the standard deviation of X?7which is the standard deviation of Y a given mass of a gas at - 73C exerts pressure of 40 cm of mercury. what pressure will be exerted at 127 C if the volume remains constant Two planes at started to take off at the same time. Plane One took off from the tarmac going 375 mph at an angle of elevation of 40 and Plane Two took off from the tarmac going 400 mph at an angle of elevation of 35. Which plane is rising faster? What would you change to make sure both planes rise at the same rate? A solution containing 30 g of potassium chlorate in 100 mL of water at 70 C isSupersaturated MagasaturadeUnsaturated Saturated 1 point The following expression represents monthly growth. What is the yearly rate of growth? (1.04)^m Read the passage.Janelle could not believe her eyes when the cast list for the spring play was finally posted. Not only was she cast in the role she wanted, but her brother was cast in the leading role!Which best describes an example of the authors use of diction in the passage?The idiom could not believe her eyes emphasizes Janelles amazement.The phrase not only emphasizes that Janelle is upset about the list.The repetition of the word cast suggests the play is important to Janelle.The phrase leading role suggests that Janelle is proud of her brother. PLEASE SOMEONE HELP ASAP WILL GIVE ANYTHING(Theoretical Probability MC)Determine P(not yellow) if the spinner is spun once. 75% 37.5% 25% 12.5% Jack paid $46 for 4 pizzas. How much did he pay per pizza? Explain how ancient glacial deposits in Africa, India, Australia, and South America support the idea of continental drift . Charlie is saving money to buy a game. So far he has saved $20, which is one-fourth of the total cost of the game. how much does the game cost? Objects with masses M1=12.0 kg and M2= 7.0 kg are connected by a light string that passes over a frictionless pulley as in the figure below. If, when the system starts from rest, M2 falls 1.00m in 1.33s, determine the coefficient of Kinect friction between M1 and the table For a standard normal distribution, find:P(Z > -1.79)Express the probability as a decimal rounded to 4 decimalplaces.